Data Sufficiency Questions for MAH – CET | Download PDF

0
322
Data Sufficiency for MAH-CET 2022
Data Sufficiency for MAH-CET 2022

Data Sufficiency Questions for MAH – CET 2022 – Download PDF

Here you can download CMAT 2022 – important MAH – CET Data Sufficiency Questions PDF by Cracku. Very Important MAH – CET 2022 and These questions will help your MAH – CET preparation. So kindly download the PDF for reference and do more practice.

Download Data Sufficiency Questions for MAH – CET

Enroll to MAH-CET Crash Course

Instructions

Each of the questions below consists of a question and two statements numbered I and II given below it. You have to decide whether the data provided in the statement are sufficient to answer the question. Read both the statements and give answer.

Question 1: On which day of the week did Sohan visit Chennai ?
a: Sohan returned to Hyderabad from Chennai on Thursday
b: Sohan left Hyderabad on Monday for Chennai.

a) If the data in statement I alone are sufficient to answer the question, while the statement II alone are not sufficient to answer the question.

b) If the data in statement II alone are sufficient to answer the question, while the statement I alone are not sufficient to answer the question.

c) If the data either in statement I alone or in statement II alone are sufficient to answer the question.

d) If the data given in both the statements I and II together are not sufficient to answer the question

e) If the data in both the statements I and II together are necessary to answer the question.

Question 2: Towards which direction is Q from R ?
a: Q is exactly to the east of M
b: M is exactly to the south of R.

a) If the data in statement I alone are sufficient to answer the question, while the statement II alone are not sufficient to answer the question.

b) If the data in statement II alone are sufficient to answer the question, while the statement I alone are not sufficient to answer the question.

c) If the data either in statement I alone or in statement II alone are sufficient to answer the question.

d) If the data given in both the statements I and II together are not sufficient to answer the question

e) If the data in both the statements I and II together are necessary to answer the question.

Question 3: Who amongst P, Q, R, S and T is the heaviest ?
a: Q is heavier than R and T but lighter than only S.
b: R is third from the top when they are arranged in descending order of their weight and is heavier than T and P.

a) If the data in statement I alone are sufficient to answer the question, while the statement II alone are not sufficient to answer the question.

b) If the data in statement II alone are sufficient to answer the question, while the statement I alone are not sufficient to answer the question.

c) If the data either in statement I alone or in statement II alone are sufficient to answer the question.

d) If the data given in both the statements I and II together are not sufficient to answer the question

e) If the data in both the statements I and II together are necessary to answer the question.

Question 4: How is ‘then’ written in a code language ?
a: ‘now and then’ is written as ‘ka da ta’ in that code language
b: ‘then you come’ is written as ‘da ma pa’ in that code language

a) If the data in statement I alone are sufficient to answer the question, while the statement II alone are not sufficient to answer the question.

b) If the data in statement II alone are sufficient to answer the question, while the statement I alone are not sufficient to answer the question.

c) If the data either in statement I alone or in statement II alone are sufficient to answer the question.

d) If the data given in both the statements I and II together are not sufficient to answer the question

e) If the data in both the statements I and II together are necessary to answer the question.

Question 5: How many daughters does M have ?
a: P and Q are the only daughters of N.
b: D is brother of P and son of M. D and Q are siblings.

a) If the data in statement I alone are sufficient to answer the question, while the statement II alone are not sufficient to answer the question.

b) If the data in statement II alone are sufficient to answer the question, while the statement I alone are not sufficient to answer the question.

c) If the data either in statement I alone or in statement II alone are sufficient to answer the question.

d) If the data given in both the statements I and II together are not sufficient to answer the question

e) If the data in both the statements I and II together are necessary to answer the question.

Take Free MAH-CET mock tests here

Instructions

Each of the questions below consists of a question and two or three statements given below it. You have to decide whether the data provided in the statements are sufficient to answer the question.

Question 6: What is the rank of Suresh in the class of 17 students ?
1. Nitin, having 13th rank from the bottom, is six ranks ahead of Bhupesh, who is two ranks behind Suresh.
2. Bhupesh is four ranks ahead of Kamlesh
3. Bhupesh is two ranks behind Suresh and Kamlesh’s rank is 15.

a) Only 1 alone is sufficient

b) Either 1 alone or 2 and 3 together are sufficient

c) Only 2 and 3 together are sufficient

d) Only 1 and 3 together are sufficient

e) None of these

Question 7: How much did Sohil get as profit at the year end in the business done by Animesh, Sohil and Akhilesh ?
1. Akhilesh invested Rs. 8000/- for nine months, his profit was (3/2) times that of Sohil’s and his investment was 4 times that of Animesh.
2. Animesh and Sohil invested for one year and in the proportion 1 : 2 respectively.
3. The three together got Rs. 1000/- as profit at the year end.

a) Only 1 and 2 together are sufficient

b) Only 1 and 3 together are sufficient

c) 1, 2 and 3 together are not sufficient

d) 1, 2 and 3 together are necessary

e) None of these

Instructions

Each of the questions below consists of a question and two statements numbered I and II given below it. You have to decide whether the data provided in the statements are sufficient to
answer the question. Read both the statements and answer the below questions

a: if the data in Statement I alone are sufficient to answer the question, while the data in Statement II alone are not sufficient to answer the question.
b: if the data in Statement II alone are sufficient to answer the question, while the data in Statement I alone are not sufficient to answer the question.
c: if the data either in Statement I alone or in Statement II alone are sufficient to answer the question.
d: if the data even in both Statements I and II together are not sufficient to answer the question.
e: if the data in both Statements I and II together are necessary to answer the question.

Question 8: How is A related to F?I. A is mother of B. D is brother of B. R is father of D. R has one son and one daughter. T is father of R. T is married to F.
II. F is married to T. T has only two children R and C. R is married to A. A has two children. C is aunt of B and D.

a) if the data in Statement I alone are sufficient to answer the question, while the data in Statement II alone are not sufficient to answer the question.

b) if the data in Statement II alone are sufficient to answer the question, while the data in Statement I alone are not sufficient to answer the question.

c) if the data either in Statement I alone or in Statement II alone are sufficient to answer the question.

d) if the data even in both Statements I and II together are not sufficient to answer the question.

e) if the data in both Statements I and II together are necessary to answer the question.

Question 9: What is the code for ‘reason’ in a certain code language?
I. In that code language ‘little reason to believe’ is coded as ‘& 4 $ 2’ and ‘reason is never little’ is coded as ‘3 & 8 2’.
II. In that code language ‘little to reason now’ is coded as ‘& 2 % 4’ and ‘believe now is problem’ is coded as ‘% 8 $ 5’

a) if the data in Statement I alone are sufficient to answer the question, while the data in Statement II alone are not sufficient to answer the question.

b) if the data in Statement II alone are sufficient to answer the question, while the data in Statement I alone are not sufficient to answer the question.

c) if the data either in Statement I alone or in Statement II alone are sufficient to answer the question.

d) if the data even in both Statements I and II together are not sufficient to answer the question.

e) if the data in both Statements I and II together are necessary to answer the question.

Instructions

In each of the questions below consists of a question and two statements numbered I and II given below it. You have to decide whether the data provided in the statements are sufficient to answer the question. Read both the statements and
Give answer
(A) If the data in statement I alone are sufficient to answer the question, while the data in statement II alone are not sufficient to answer the question
(B) If the data in statement II alone are sufficient to answer the question, while the data in statement I alone are not sufficient to answer the question
(C) If the data either in statement I alone or in statement II alone are sufficient to answer the question
(D) If the data given in both statements I and II together are not sufficient to answer the question
(E) If the data in both statements I and II together are necessary to answer the question.

Question 10: Which word in the code language ‘flower’ means?
I. ‘de fu la pane’ means ‘rose flower is beautiful’ and ‘la quiz’ means ‘beautiful tree’.
II. ‘de la chin’ means ‘red rose flower’ and ‘pa chin’ means ‘red tea’.

a) If the data in statement I alone are sufficient to answer the question, while the data in statement II alone are not sufficient to answer the question

b) If the data in statement II alone are sufficient to answer the question, while the data in statement I alone are not sufficient to answer the question

c) If the data either in statement I alone or in statement II alone are sufficient to answer the question

d) If the data given in both statements I and II together are not sufficient to answer the question

e) If the data in both statements I and II together are necessary to answer the question.

Question 11: How is  P is related to the Q?
I. J has the two daughters ,one of them R is married to P.
II. Q is the mother of S, who is the younger sister of R.

a) If the data in statement I alone are sufficient to answer the question, while the data in statement II alone are not sufficient to answer the question

b) If the data in statement II alone are sufficient to answer the question, while the data in statement I alone are not sufficient to answer the question

c) If the data either in statement I alone or in statement II alone are sufficient to answer the question

d) If the data given in both statements I and II together are not sufficient to answer the question

e) If the data in both statements I and II together are necessary to answer the question.

Question 12: Among P,Q,R,S and T, who ranks third in the terms of salary obtained by them?
I. T’s salary is more than P and Q but not more than S.
II. R’s salary is lowest among them.

a) If the data in statement I alone are sufficient to answer the question, while the data in statement II alone are not sufficient to answer the question

b) If the data in statement II alone are sufficient to answer the question, while the data in statement I alone are not sufficient to answer the question

c) If the data either in statement I alone or in statement II alone are sufficient to answer the question

d) If the data given in both statements I and II together are not sufficient to answer the question

e) If the data in both statements I and II together are necessary to answer the question.

Question 13: Can Rohan retire from office X in the April 2000, with full pension benefits?
I. Rohan will completes 30 years of service in office X in April 2000 and desires to retire.
II. As per office X rules an employee has to complete 30 years of service and attain age of 60 years. Rohan was 3 years to complete age of 60 years.

a) If the data in statement I alone are sufficient to answer the question, while the data in statement II alone are not sufficient to answer the question

b) If the data in statement II alone are sufficient to answer the question, while the data in statement I alone are not sufficient to answer the question

c) If the data either in statement I alone or in statement II alone are sufficient to answer the question

d) If the data given in both statements I and II together are not sufficient to answer the question

e) If the data in both statements I and II together are necessary to answer the question.

Question 14: Which village is to the North-East of village ‘A’.?
I. Village B is to the North of village A, villages C and D are to the East and West of the of the village B respectively.
II. Village P is to the South of Village A and village E is to the East of the village P. village K is to the North of village P.

a) If the data in statement I alone are sufficient to answer the question, while the data in statement II alone are not sufficient to answer the question

b) If the data in statement II alone are sufficient to answer the question, while the data in statement I alone are not sufficient to answer the question

c) If the data either in statement I alone or in statement II alone are sufficient to answer the question

d) If the data given in both statements I and II together are not sufficient to answer the question

e) If the data in both statements I and II together are necessary to answer the question.

Instructions

Each of the following question consists of a question and two statements numbered I and II given below it.You have to decide whether the data provided the statement are sufficient to answer the question Read the statements and Give answer

a:If the data in Statement I alone are sufficient to answer the question while the data in Statement II alone are not Sufficient to answer the question
b:If the data in Statement II alone are sufficient to answer the question While the data in Statement I alone are not Sufficient to answer the question
c:If the data either in Statement I alone or in Statement II alone are sufficient to answer the question
d:If the data in both the statements I and II together are not Sufficient to answer the question
e:If the data in both the Statements I and II together are necessary to answer the question

Question 15: How is ‘go’ written in code language ?
I.’go over there’ is written as ‘pa da na’ in that code language.
II.’go and sit’ is written as ‘sa ka pa’ in that code language

a) If the data in Statement I alone are sufficient to answer the question while the data in Statement II alone are not Sufficient to answer the question

b) If the data in Statement II alone are sufficient to answer the question While the data in Statement I alone are not Sufficient to answer the question

c) If the data either in Statement I alone or in Statement II alone are sufficient to answer the question

d) If the data in both the statements I and II together are not Sufficient to answer the question

e) If the data in both the Statements I and II together are necessary to answer the question

Question 16: In a row of thirty students facing North, what is R’s position from the left end ?
I.There are twelve students between R and Q
II.T is tenth from the right end there are sixteen students between T and R.

a) If the data in Statement I alone are sufficient to answer the question while the data in Statement II alone are not Sufficient to answer the question

b) If the data in Statement II alone are sufficient to answer the question While the data in Statement I alone are not Sufficient to answer the question

c) If the data either in Statement I alone or in Statement II alone are sufficient to answer the question

d) If the data in both the statements I and II together are not Sufficient to answer the question

e) If the data in both the Statements I and II together are necessary to answer the question

Question 17: How many sister does K have ?
I. M is sister of K
II. K’s mother has three children

a) If the data in Statement I alone are sufficient to answer the question while the data in Statement II alone are not Sufficient to answer the question

b) If the data in Statement II alone are sufficient to answer the question While the data in Statement I alone are not Sufficient to answer the question

c) If the data either in Statement I alone or in Statement II alone are sufficient to answer the question

d) If the data in both the statements I and II together are not Sufficient to answer the question

e) If the data in both the Statements I and II together are necessary to answer the question

Question 18: Who among A,B,C, D and E, each having a different heights is the third tallest ?
I.E is shorter than only B
II. C is taller than only A

a) If the data in Statement I alone are sufficient to answer the question while the data in Statement II alone are not Sufficient to answer the question

b) If the data in Statement II alone are sufficient to answer the question While the data in Statement I alone are not Sufficient to answer the question

c) If the data either in Statement I alone or in Statement II alone are sufficient to answer the question

d) If the data in both the statements I and II together are not Sufficient to answer the question

e) If the data in both the Statements I and II together are necessary to answer the question

Question 19: Village P is towards which directions of village R ?
I. Village R is to the South-East of village T and T is to the North of village P.
II. Village Q is the South of village P and to the South-West of village R.

a) If the data in Statement I alone are sufficient to answer the question while the data in Statement II alone are not Sufficient to answer the question

b) If the data in Statement II alone are sufficient to answer the question While the data in Statement I alone are not Sufficient to answer the question

c) If the data either in Statement I alone or in Statement II alone are sufficient to answer the question

d) If the data in both the statements I and II together are not Sufficient to answer the question

e) If the data in both the Statements I and II together are necessary to answer the question

Instructions

Each of the questions below consists of a question and two statements numbered I and II given below it. You have to decide whether the data provided in the statements are sufficient to
answer the question. Read both the statements and answer the below questions

a: if the data in Statement I alone are sufficient to answer the question, while the data in Statement II alone are not sufficient to answer the question.
b: if the data in Statement II alone are sufficient to answer the question, while the data in Statement I alone are not sufficient to answer the question.
c: if the data either in Statement I alone or in Statement II alone are sufficient to answer the question.
d: if the data even in both Statements I and II together are not sufficient to answer the question.
e: if the data in both Statements I and II together are necessary to answer the question.

Question 20: How is A related to F?I. A is mother of B. D is brother of B. R is father of D. R has one son and one daughter. T is father of R. T is married to F.
II. F is married to T. T has only two children R and C. R is married to A. A has two children. C is aunt of B and D.

a) if the data in Statement I alone are sufficient to answer the question, while the data in Statement II alone are not sufficient to answer the question.

b) if the data in Statement II alone are sufficient to answer the question, while the data in Statement I alone are not sufficient to answer the question.

c) if the data either in Statement I alone or in Statement II alone are sufficient to answer the question.

d) if the data even in both Statements I and II together are not sufficient to answer the question.

e) if the data in both Statements I and II together are necessary to answer the question.

Get 5 MAH-CET mocks at just Rs.299

Answers & Solutions:

1) Answer (D)

Since, we do not have any data about the time required to travel to Chennai even after combining I and II.

Thus, I and II together are not sufficient to answer the question.

2) Answer (E)

Clearly, I alone & II alone are insufficient as there is no info about R and Q in statements I and II respectively.

I & II : Q is to the east of M and M is to the south of R.

=> Q is to the south-east of R.

Thus, I & II together are sufficient.

3) Answer (A)

I : Q is lighter than only S. This means Q is the heaviest.

Thus, I alone is sufficient.

II : R is third heaviest and is also heavier than T & P.

=> S, Q > R > T, P

We cannot conclude who is heavier between S and Q.

Thus II alone is not sufficient.

4) Answer (E)

Since “then” is the only word present in both the sentences, its code can be uniquely determined. Hence, both the statements are necessary to solve the question.

then – Da

Option E is the right answer.

5) Answer (E)

Using statement a, we know that P and Q are only daughters of N. But we cannot establish any relationship between N and M. Hence, statement a alone is insufficient.

Using statement b, we know that D,Q and P are the children of M. Bot we do not know whether Q is a male or female member. Hence, statement b alone is insufficient.

Using both statements A and B,we can conclude that D,P and Q are siblings and they are children of M and N. Also, it is clear that P and Q are the only daughters of M and N (from statement a). Hence, both the statements together are necessary to solve the question. Option E is the right answer.

6) Answer (B)

Total students in class = 17

1 : Nitin, having 13th rank from the bottom, is six ranks ahead of Bhupesh, who is two ranks behind Suresh.

=> Nitin’s rank from top = (17+1)-13 = 5th

Thus, Bhupesh’s rank from top = 11th

=> Suresh’ rank = 11-2 = 9th

Clearly, 2 alone or 3 alone are insufficient as there is not enough data, thus by combining them, we get :

Kamlesh’s rank = 15th

Thus, Bhupesh’s rank from top = 11th

=> Suresh’ rank = 11-2 = 9th

Thus, either 1 alone or 2 and 3 together are sufficient.

=> Ans – (B)

7) Answer (D)

Clearly, each statement alone is not sufficient, thus combining all the three statements, we get :

Amount invested by Akhilesh = Rs. 8000 for 9 months

=> Amount invested by Animesh = Rs. 2000 for 1 year

=> Amount invested by Sohil = Rs. 4000 for 1 year

Thus, ratio of profit of Akhilesh, Animesh and Sohil = $(8000\times9):(2000\times12):(4000\times12)$

= $72:24:48=3:1:2$

Total profit = Rs. 1000

Profit of Sohil = $\frac{2}{(3+1+2)}\times1000=Rs.$ $333.33$

Thus, 1, 2 and 3 together are necessary.

=> Ans – (D)

8) Answer (A)

I : A is mother of B. D is brother of B. R is father of D. R has one son and one daughter.

=> R is husband of A, and B is sister of D.

T is father of R. T is married to F.

=> F is mother of R.

Thus, A is daughter-in-law of F, hence I alone is sufficient.


II : F is married to T. T has only two children R and C.

R is married to A. A has two children. C is aunt of B and D.

Thus, A can be either son-in-law or daughter-in-law of F, hence II alone is sufficient.

$\therefore$ Statement I alone is sufficient.

=> Ans – (A)

9) Answer (D)

I : In that code language ‘little reason to believe’ is coded as ‘& 4 $ 2’ and ‘reason is never little’ is coded as ‘3 & 8 2’.

The common words in both inferences are ‘little‘ and ‘reason‘ coded as = ‘&’ or ‘2’

Thus, we cannot find the code for reason, hence I alone is insufficient.

II : In that code language ‘little to reason now’ is coded as ‘& 2 % 4’ and ‘believe now is problem’ is coded as ‘% 8 $ 5

The only common word is ‘now‘ coded as = ‘%’

Thus, we cannot find the code for reason, hence II alone is insufficient.

Similarly, by combining both statements, we still cannot find the code for ‘reason’.

$\therefore$ Both statements even together are not sufficient.

=> Ans – (D)

10) Answer (D)

I : ‘de fu la pane’ means ‘rose flower is beautiful’ and ‘la quiz’ means ‘beautiful tree’.

The only common word in above inferences is ‘beautiful‘ coded as = ‘la’

Thus, we cannot find the code of ‘flower’ from I alone.


II : ‘de la chin’ means ‘red rose flower’ and ‘pa chin’ means ‘red tea’.

The only common word in above inferences is ‘red‘ coded as = ‘chin’

Thus, we cannot find the code of ‘flower’ from II alone.


I & II : In the above statements, common words are ‘rose‘ and ‘flower‘ coded as = ‘de’ or ‘la’

Thus, code for flower is either ‘de’ or ‘la’.

$\therefore$ Both statements even together are insufficient.

=> Ans – (D)

11) Answer (E)

Clearly, both statements alone are insufficient as information about Q and P are not given in respective statements.

J has the two daughters, one of them R is married to P.

Q is the mother of S, who is the younger sister of R, => J is husband of Q.

Thus, P is son-in-law of Q.

$\therefore$ Both statements together are sufficient.

=> Ans – (E)

12) Answer (D)

I : T’s salary is more than P and Q but not more than S.

=> $S > T > P, Q$

There is no information about R, hence we cannot find the person with third highest salary by I alone.

Similarly, statement II alone is also insufficient.

R’s salary is lowest among them.

=> $S>T>P,Q>R$

Thus, the person with third highest salary can be either P or Q.

$\therefore$ Both statements even together are insufficient.

=> Ans – (D)

13) Answer (E)

From first statement, we do not know the rules about full pension benefits of the company, and from the second statement, we do not know how long Rohan has worked for the company, thus both statements alone are not sufficient.

Combining both statements, we get that Rohan will complete 30 years of service and will attain age of 60 years, hence is eligible for full pension benefits.

$\therefore$ Both statements together are sufficient.

=> Ans – (E)

14) Answer (A)

I : Village B is to the North of village A, villages C and D are to the East and West of the of the village B respectively.

Thus, village C is to the north-east of village A, hence I alone is sufficient.


II : Village P is to the South of Village A and village E is to the East of the village P. village K is to the North of village P.

We do not know whether village K is to the north or south of A, hence II alone is not sufficient.

$\therefore$ Statement I alone is sufficient.

=> Ans – (A)

15) Answer (E)

Clearly, I alone & II alone is insufficient.

I & II : Combining I & II, the common word is go.

=> code for go = ‘pa’

Thus, I & II together are sufficient.

16) Answer (B)

I : We do not know the position of Q. Hence, we can’t determine the position of R.

Thus, I alone is insufficient.


II : T is tenth from right end in a row of 30 students.

=> T’s position from left end = 30+1-10 = 21st

Since, there are 16 students between T and R, => R is to the left of T.

Position of R from left end = 4th.

Thus, II alone is sufficient.

17) Answer (D)

I : We do not know about any other children. Thus, I alone is insufficient.

II : We do not know the gender of the three children. Thus, II alone is insufficient.

I & II : Combining I & II, we get that K has 2 other siblings, one being M(female).

We do not know the gender of the 2nd sibling.

Thus, I & II even together are not sufficient.

18) Answer (E)

I : E is shorter than only B.

=> B is the tallest and E is the second tallest. => B > E > A, C, D.

Since, we can’t find the third tallest, I alone is insufficient.

II : C is taller than only A.

=> A is the shortest and C is the fourth tallest. => B,D,E > C > A

Since, we can’t find the third tallest, II alone is insufficient.


I & II : From I and II, we get

B > E > D > C > A.

=> D is the third tallest.

Thus, I & II together are sufficient.

19) Answer (D)

I : Village R is south east of Village T and there is no specific information about position of village P.

Thus, I alone is insufficient.

Similarly, in II, there is no specific information about position of R.

Thus, II alone is insufficient.


I & II : We know, that T is north of P and Q is south of P. Thus, T, P & Q are in a straight vertical line in that order.

There is still not specific info about position of P. P can be to the west or north-west or south-west of R.

Thus, I & II together are not sufficient.

20) Answer (A)

I : A is mother of B. D is brother of B. R is father of D. R has one son and one daughter.

=> R is husband of A, and B is sister of D.

T is father of R. T is married to F.

=> F is mother of R.

Thus, A is daughter-in-law of F, hence I alone is sufficient.


II : F is married to T. T has only two children R and C.

R is married to A. A has two children. C is aunt of B and D.

Thus, A can be either son-in-law or daughter-in-law of F, hence II alone is sufficient.

$\therefore$ Statement I alone is sufficient.

=> Ans – (A)

Take MAH-CET Mock Tests

Enroll to CAT 2022 course

LEAVE A REPLY

Please enter your comment!
Please enter your name here